Difference between revisions of "2025 AIME II Problems/Problem 6"
(→Problem) |
(→Solution 2 (faster)) |
||
Line 51: | Line 51: | ||
<math>[EFGH]=2x\cdot2y=2\cdot\frac{6}{\sqrt{5}}\cdot2\cdot\frac{12}{\sqrt{5}}=\frac{288}{5}</math>, so the answer is <math>288+5=\boxed{293}</math>. ~eevee9406 | <math>[EFGH]=2x\cdot2y=2\cdot\frac{6}{\sqrt{5}}\cdot2\cdot\frac{12}{\sqrt{5}}=\frac{288}{5}</math>, so the answer is <math>288+5=\boxed{293}</math>. ~eevee9406 | ||
− | == Solution 2 ( | + | == Solution 2 (Faster) == |
Denote the intersection of <math>BC</math> and <math>w_1</math> as <math>P</math>, the intersection of <math>BC</math> and <math>GH</math> be <math>Q</math>, and the center of <math>w_2</math> to be <math>O</math>. Additionally, let <math>EF = GH = a, FG = EH = b</math>. We have that <math>CP = 18</math> and <math>PQ = \frac{6-b}{2}</math>. Considering right triangle <math>OAD</math>, <math>AD = 12</math>. Letting <math>R</math> be the intersection of <math>AD</math> and <math>FG</math>, <math>DR = 12 - \frac{b}{2}</math>. Using the equivalent area ratios: <cmath> \frac{a(24-\frac{b}{2})}{2} = \frac{(12-\frac{a}{2})b}{2} </cmath> | Denote the intersection of <math>BC</math> and <math>w_1</math> as <math>P</math>, the intersection of <math>BC</math> and <math>GH</math> be <math>Q</math>, and the center of <math>w_2</math> to be <math>O</math>. Additionally, let <math>EF = GH = a, FG = EH = b</math>. We have that <math>CP = 18</math> and <math>PQ = \frac{6-b}{2}</math>. Considering right triangle <math>OAD</math>, <math>AD = 12</math>. Letting <math>R</math> be the intersection of <math>AD</math> and <math>FG</math>, <math>DR = 12 - \frac{b}{2}</math>. Using the equivalent area ratios: <cmath> \frac{a(24-\frac{b}{2})}{2} = \frac{(12-\frac{a}{2})b}{2} </cmath> | ||
− | This equation gives <math>b=2a</math>. Using | + | This equation gives <math>b=2a</math>. Using the Pythagorean Theorem on triangle <math>GHE</math> gives that <math>a^2+b^2 = 144</math>. Plugging the reuslt <math>b=2a</math> into this equation gives that the area of the triangle is <math>\frac{288}{5} \to \boxed{293}</math>. |
− | + | ~ Vivdax | |
+ | |||
+ | ~ Edited by [https://artofproblemsolving.com/wiki/index.php/User:Aoum aoum] | ||
==See also== | ==See also== |
Revision as of 16:33, 14 February 2025
Problem
Circle with radius
centered at point
is internally tangent at point
to circle
with radius
. Points
and
lie on
such that
is a diameter of
and
. The rectangle
is inscribed in
such that
,
is closer to
than to
, and
is closer to
than to
, as shown. Triangles
and
have equal areas. The area of rectangle
is
, where
and
are relatively prime positive integers. Find
.
Solution 1 (thorough)
Let and
. Notice that since
is perpendicular to
(can be proven using basic angle chasing) and
is an extension of a diameter of
, then
is the perpendicular bisector of
. Similarly, since
is perpendicular to
(also provable using basic angle chasing) and
is part of a diameter of
, then
is the perpendicular bisector of
.
From the Pythagorean Theorem on , we have
, so
. To find our second equation for our system, we utilize the triangles given.
Let . Then we know that
is also a rectangle since all of its angles can be shown to be right using basic angle chasing, so
. We also know that
.
and
, so
. Notice that
is a height of
, so its area is
.
Next, extend past
to intersect
again at
. Since
is given to be a diameter of
and
, then
is the perpendicular bisector of
; thus
. By Power of a Point, we know that
.
and
, so
and
.
Denote . We know that
(recall that
, and it can be shown that
is a rectangle).
is the height of
, so its area is
.
We are given that (
denotes the area of figure
). As a result,
. This can be simplified to
. Substituting this into the Pythagorean equation yields
and
. Then
.
, so the answer is
. ~eevee9406
Solution 2 (Faster)
Denote the intersection of and
as
, the intersection of
and
be
, and the center of
to be
. Additionally, let
. We have that
and
. Considering right triangle
,
. Letting
be the intersection of
and
,
. Using the equivalent area ratios:
This equation gives . Using the Pythagorean Theorem on triangle
gives that
. Plugging the reuslt
into this equation gives that the area of the triangle is
.
~ Vivdax
~ Edited by aoum
See also
2025 AIME II (Problems • Answer Key • Resources) | ||
Preceded by Problem 5 |
Followed by Problem 7 | |
1 • 2 • 3 • 4 • 5 • 6 • 7 • 8 • 9 • 10 • 11 • 12 • 13 • 14 • 15 | ||
All AIME Problems and Solutions |
The problems on this page are copyrighted by the Mathematical Association of America's American Mathematics Competitions.